LSAT and Law School Admissions Forum

Get expert LSAT preparation and law school admissions advice from PowerScore Test Preparation.

User avatar
 Dave Killoran
PowerScore Staff
  • PowerScore Staff
  • Posts: 5853
  • Joined: Mar 25, 2011
|
#22272
Complete Question Explanation
(The complete setup for this game can be found here: lsat/viewtopic.php?t=15690)

The correct answer choice is (B)

If T delivers the third speech, then from the last rule J cannot deliver the second or fourth speech. When this information is combined with the fourth rule prohibiting J from giving the first and last speech, we can infer that J must give the fifth speech. Consequently, answer choice (B) is correct.

Note that this question reflects one of the inferences drawn during the game setup.

Get the most out of your LSAT Prep Plus subscription.

Analyze and track your performance with our Testing and Analytics Package.